Last visit was: 26 Apr 2024, 14:51 It is currently 26 Apr 2024, 14:51

Close
GMAT Club Daily Prep
Thank you for using the timer - this advanced tool can estimate your performance and suggest more practice questions. We have subscribed you to Daily Prep Questions via email.

Customized
for You

we will pick new questions that match your level based on your Timer History

Track
Your Progress

every week, we’ll send you an estimated GMAT score based on your performance

Practice
Pays

we will pick new questions that match your level based on your Timer History
Not interested in getting valuable practice questions and articles delivered to your email? No problem, unsubscribe here.
Close
Request Expert Reply
Confirm Cancel
SORT BY:
Date
Tags:
Show Tags
Hide Tags
avatar
Intern
Intern
Joined: 16 Mar 2013
Posts: 13
Own Kudos [?]: 27 [10]
Given Kudos: 6
Location: United States
Concentration: International Business, Entrepreneurship
Schools: NTU '15 (S)
GPA: 3.08
WE:Marketing (Manufacturing)
Send PM
Most Helpful Reply
Director
Director
Joined: 03 Feb 2011
Status:Retaking after 7 years
Posts: 860
Own Kudos [?]: 4468 [5]
Given Kudos: 221
Location: United States (NY)
Concentration: Finance, Economics
GMAT 1: 720 Q49 V39
GPA: 3.75
Send PM
General Discussion
User avatar
Manager
Manager
Joined: 04 Apr 2013
Posts: 96
Own Kudos [?]: 177 [4]
Given Kudos: 36
Send PM
avatar
Intern
Intern
Joined: 16 Mar 2013
Posts: 13
Own Kudos [?]: 27 [0]
Given Kudos: 6
Location: United States
Concentration: International Business, Entrepreneurship
Schools: NTU '15 (S)
GPA: 3.08
WE:Marketing (Manufacturing)
Send PM
Re: Reporting a 90 percent drop in net income during the second quarter [#permalink]
Marcab wrote:
pjaseem wrote:
Reporting a 90 percent drop in net income during the second quarter, dragged down by restructuring charges and weak sales, the earnings guidance for the year was withdrawn by Best Buy Co.

A.Reporting a 90 percent drop in net income during the second quarter, dragged down by restructuring charges and weak sales, the earnings guidance for the year was withdrawn by Best Buy Co.
B.Best Buy Co. is reporting a 90 percent drop in net income during the second quarter, dragged down by restructuring charges and weak sales; the company also withdrew its earnings guidance for the year.
C.Reporting a 90 percent drop in net income during the second quarter dragged down by restructuring charges and weak sales, Best Buy Co. withdrew the earnings guidance for the year.
D.Best Buy Co. has reported a 90 percent drop in net income during the second quarter, dragged down by restructuring charges and weak sales; the company has also withdrawn its earnings guidance for the year.
E.Best Buy Co. is reporting a 90 percent drop in net income during the second quarter, dragged down by restructuring charges and weak sales; the company has also withdrawn its earnings guidance for the year.


Please do mention the source.
Who is reporting? What has brought down the percent income? In retaliation, what has Best Buy done?
These are the questions that should be answered by the correct answer choice.
A has modifier issue, C has merged two parts of the sentences into one without any verb or any conjunction.
B and E use present continuous, which is really not needed.
D is the ideal choice here.
Regards




The source is 4gmat blog.

I think the answer choice D is incorrect.The modifier "dragged down by restructuring charges" is not close to the noun it modifies .Morover the modifier doesn't seem to be an adverbial clause. Experts correct me if i am wrong.
avatar
Intern
Intern
Joined: 16 Mar 2013
Posts: 13
Own Kudos [?]: 27 [0]
Given Kudos: 6
Location: United States
Concentration: International Business, Entrepreneurship
Schools: NTU '15 (S)
GPA: 3.08
WE:Marketing (Manufacturing)
Send PM
Re: Reporting a 90 percent drop in net income during the second quarter [#permalink]
maaadhu wrote:
pjaseem wrote:
Reporting a 90 percent drop in net income during the second quarter, dragged down by restructuring charges and weak sales, the earnings guidance for the year was withdrawn by Best Buy Co.

A.Reporting a 90 percent drop in net income during the second quarter, dragged down by restructuring charges and weak sales, the earnings guidance for the year was withdrawn by Best Buy Co.
B.Best Buy Co. is reporting a 90 percent drop in net income during the second quarter, dragged down by restructuring charges and weak sales; the company also withdrew its earnings guidance for the year.
C.Reporting a 90 percent drop in net income during the second quarter dragged down by restructuring charges and weak sales, Best Buy Co. withdrew the earnings guidance for the year.
D.Best Buy Co. has reported a 90 percent drop in net income during the second quarter, dragged down by restructuring charges and weak sales; the company has also withdrawn its earnings guidance for the year.
E.Best Buy Co. is reporting a 90 percent drop in net income during the second quarter, dragged down by restructuring charges and weak sales; the company has also withdrawn its earnings guidance for the year.



None of the options seems correct.

As per e-gmat explanation, "ed modifier" always modifies the preceding noun or noun phrase. So in all the options, "dragged down" is modifying second quarter. However, its not the second quarter that is dragged down but the net income is dragged down.

Below are the examples are given by e-gmat. Experts can weigh in if I am wrong.

Verb-ed Modifiers
1: Always modify preceding noun or noun phrase.

Discouraged by the long hours and low pay, my sister finally quit her job. Correct.

My sister, discouraged by the long hours and low pay, finally quit her job. Correct.

My sister finally quit her job, discouraged by the long hours and low pay. Incorrect as per GMAT rules Here the verb-ed modifier is modifying the preceding noun “her job” which does not make sense.



I agree. :) . the answer choice D seems to be wrong
SVP
SVP
Joined: 06 Nov 2014
Posts: 1798
Own Kudos [?]: 1368 [2]
Given Kudos: 23
Send PM
Re: Reporting a 90 percent drop in net income during the second quarter [#permalink]
2
Kudos
Expert Reply
Things to keep in mind:

1. Reporting has already been done - past tense for reporting
2. Best Buy Co. is doing the reporting. - correct modifier. We need to tell correctly who did the reporting

Option A can be eliminated based on the 2nd point.
Option B uses a present continuous tense for reporting. Hence incorrect
Option C merges two parts of the sentence without using any conjunction
Option E uses a present continuous tense for reporting. Hence incorrect

Correct Option: D
Although there are issues with this option too, but this is the best of the lot.
Retired Moderator
Joined: 10 Mar 2013
Status:The best is yet to come.....
Posts: 397
Own Kudos [?]: 832 [2]
Given Kudos: 235
Send PM
Re: Reporting a 90 percent drop in net income during the second quarter [#permalink]
2
Kudos
OE

https://practice-questions.wizako.com/g ... se-2.shtml

Step 1: Identifying What Is Tested

Look for differences across the answer options to identify what is being tested in the sentence.
Some of the options use “reporting a 90%.... and weak sales” as a modifier in the sentence and some of the options use it as a main part of the sentence.
When this phrase is part of the main part of the sentence, the tense construction varies with some options using the progressive tense “is reporting” and some using the present perfect “has reported”.
Similarly, the verb “withdraw” has also been constructed in different tenses.

Step 2: Understanding the Sentence

The use of the progressive tense “is reporting” would imply that the company is right now doing the action of reporting and that it is still happening. Then progressive tense would therefore, be incorrect.
The tense form for “withdraw” must match the tense for “reporting” (if “reporting…” has not been used as a modifier) because the actions are related and happened at the same time.
If “reporting a 90%.... and weak sales” is used as a modifier, it has to refer to the company because the company is doing the action of reporting.

Step 3: Eliminating Options

Choice (A) is incorrect because the modifier “reporting a 90%.... and weak sales” refers to “the earnings guidance” and not to “Best Buy Co.”.
Choices (B) and (E) can be eliminated for the use of the progressive tense “is reporting”. Also, in both sentences, the verbs “withdrew” and “has withdrawn” imply that the action of withdrawing is completed. However, the verb “is reporting” leads to the impression that the action is yet to happen. This conflict in verb tense causes ambiguity in meaning.
Choice (C) uses the construction “reporting… and dragged down”. The usage of the conjunction “and” implies that there are two reasons for the withdrawal of the earnings guideline. However, this construction does not make sense because the company cannot be dragged down by restructuring charges. Rather, it is the “net income” that is dragged down. The change in the intent of the sentence is the reason to eliminate the option.

Choice D is the correct answer.
Intern
Intern
Joined: 23 Aug 2016
Posts: 21
Own Kudos [?]: 7 [0]
Given Kudos: 37
Send PM
Re: Reporting a 90 percent drop in net income during the second quarter [#permalink]
You can check out Wizako's explanation on youtube for this qs. The company has explained SC examples (including this one) well.
Current Student
Joined: 13 Apr 2019
Posts: 237
Own Kudos [?]: 65 [0]
Given Kudos: 309
Location: India
GMAT 1: 710 Q49 V36
GPA: 3.85
Send PM
Re: Reporting a 90 percent drop in net income during the second quarter [#permalink]
VeritasKarishma GMATNinja could you please explain how to eliminate C over D?
Intern
Intern
Joined: 21 Dec 2020
Posts: 41
Own Kudos [?]: 17 [0]
Given Kudos: 29
Location: India
GMAT 1: 720 Q49 V38
GPA: 3.64
Send PM
Re: Reporting a 90 percent drop in net income during the second quarter [#permalink]
How is option D correct? It says the company 'has withdrawn' its earnings guidance for the year, however that is an action that is completed in the past. How can we attach a present perfect tense to the act of withdrawal? It's not as if the company is continuing to withdraw its earnings guidance for the year even now.
User avatar
Non-Human User
Joined: 01 Oct 2013
Posts: 17226
Own Kudos [?]: 848 [0]
Given Kudos: 0
Send PM
Re: Reporting a 90 percent drop in net income during the second quarter [#permalink]
Hello from the GMAT Club VerbalBot!

Thanks to another GMAT Club member, I have just discovered this valuable topic, yet it had no discussion for over a year. I am now bumping it up - doing my job. I think you may find it valuable (esp those replies with Kudos).

Want to see all other topics I dig out? Follow me (click follow button on profile). You will receive a summary of all topics I bump in your profile area as well as via email.
GMAT Club Bot
Re: Reporting a 90 percent drop in net income during the second quarter [#permalink]
Moderators:
GMAT Club Verbal Expert
6923 posts
GMAT Club Verbal Expert
238 posts

Powered by phpBB © phpBB Group | Emoji artwork provided by EmojiOne